Notifiche
Cancella tutti

[Risolto] Buona sera scusate il disturbo,potreste darmi in aiuto per questo esercizio? Grazie mille

  

2

Calcola la derivata della funzione inversa di y=f(x)=ln(x)+2x nel punto y(0)=2,sapendo che f(x) è invertibile nel suo dominio

 

Autore

Ciao @carlitosalberto_castagna,

è corretta la traccia che hai fornito? Potresti ricontrollare gentilmente?

  1. Buonasera, si la traccia è questa,i risultati sono x(0) =1 e la derivata della funzione inversa è pari ad 1/3 😥 

@carlitosalberto_castagna va bene, il dubbio è su $y(0) = 2\,,$ sicuro che sia $x = 0$? Perché credo che questo esercizio si debba risolvere utilizzando il Teorema della derivata della funzione inversa e per tali condizioni iniziali non è applicabile direttamente.

4 Risposte



3

"sapendo che f(x) è invertibile nel suo dominio" non è la mamma dell'esattezza.
Il suo dominio è l'intero asse reale x, il suo insieme di definizione è x != 0, e il suo insieme di definizione reale è x > 0: f(x) è bijettiva, quindi invertibile, solo nel suo insieme di definizione reale (non nel dominio).
------------------------------
Usando il simbolo della funzione W(z) di Lambert si ha
* f(x) = y = ln(x) + 2*x
con derivata
* f'(x) = 1/x + 2
e
* inv[f(x)] = y = W(2*e^x)/2
con derivata
* D[inv[f(x)]] = (e^x)*W(2*e^x)/(2*(W(2*e^x) + 1)*e^x)
da cui
* f(0) → - ∞
* inv[f(x)]|0 = W(2)/2 ~= 0.43
e quindi la scrittura "y(0)=2" non si capisce che significato possa mai avere.
A meno che non si tratti di y_0 = 2, quello prodotto da f(1).
In tal caso
* D[inv[f(x)]]|2 = 1/f'(1) = 1/3
Verifica
* D[inv[f(x)]]|2 = 1/2 - 1/(2*(W(2*e^2) + 1)) = 1/3
CONTROPROVA nel paragrafo "Result" al link
http://www.wolframalpha.com/input?i=convergents%5BContinuedFraction%5B1%2F2-1%2F%282*%28productlog%282*e%5E2%29%2B1%29%29%2C11%5D%5D

 



3

\[f(x) = \log{x} + 2x = 2 \iff x = 1\,.\]

\[\frac{d}{dx}f(x) = \frac{1}{x} + 2 \Bigg|_{\substack{x = 1}} = 3 \implies\]

\[(f^{-1})'(2) = \frac{1}{f'(1)} = \frac{1}{3}\,.\]

 

  1. @enrico_bufacchi Perdonami vorrei sapere perché x(0) =1

Ciao @carlitosalberto_castagna,

semplicemente perché si cerca un valore $x$ tale che $f(x) = 2\,,$ ovvero $x = 1\,$.



2
image

f(x)=2 per x=1: punto A(1,2)

f’(1)=3=m coefficiente angolare retta tangente 

1/3 il suo reciproco è la derivata della funzione inversa in A’(2,1)

quindi coefficiente angolare della retta tangente in A’ alla inversa f^(-1)



2

f(x) = ln(x) + 2x é strettamente crescente (somma di funzioni strettamente crescenti)

per cui é invertibile. Detta u(y) la funzione inversa risulta

u[ ln(x) + 2x ] = x per definizione di funzione inversa

e per la regola di derivazione delle funzioni composte

u' [ ln (x) + 2x ] * (1/x + 2) = 1

Ora ln (x) + 2x vale 2 quando x = 1

u'(ln 1 + 2*1) * (1/1 + 2) = 1

u'(2)*3 = 1

u'(2) = 1/3

 

Grafico

https://www.desmos.com/calculator/xxoqg3ditl



Risposta
SOS Matematica

4.6
SCARICA